Menu Close

let-0-lt-a-lt-b-prove-that-ln-1-a-b-ln-1-b-a-lt-ln2-2-




Question Number 80816 by ~blr237~ last updated on 06/Feb/20
let  0<a<b  prove that   ln(1+(a/b))ln(1+(b/a))< (ln2)^2
$${let}\:\:\mathrm{0}<{a}<{b}\:\:{prove}\:{that} \\ $$$$\:{ln}\left(\mathrm{1}+\frac{{a}}{{b}}\right){ln}\left(\mathrm{1}+\frac{{b}}{{a}}\right)<\:\left({ln}\mathrm{2}\right)^{\mathrm{2}} \:\: \\ $$
Commented by ~blr237~ last updated on 06/Feb/20
Sir  , i look like  at (3−4) crossing you uze  A≤B and B≥C ⇒ A≤C    ??   if not ,please explain
$${Sir}\:\:,\:{i}\:{look}\:{like}\:\:{at}\:\left(\mathrm{3}−\mathrm{4}\right)\:{crossing}\:{you}\:{uze} \\ $$$${A}\leqslant{B}\:{and}\:{B}\geqslant{C}\:\Rightarrow\:{A}\leqslant{C}\:\:\:\:??\: \\ $$$${if}\:{not}\:,{please}\:{explain} \\ $$
Commented by mind is power last updated on 07/Feb/20
mistack sorry
$${mistack}\:{sorry} \\ $$
Answered by mr W last updated on 07/Feb/20
let x=(a/b)≠1  f(x)=ln (1+x)×ln (1+(1/x))  f′(x)=(1/(1+x))ln (1+(1/x))−(1/x^2 )×(1/(1+(1/x)))ln (1+x)  f′(x)=(1/(x(1+x)))[xln (1+(1/x))−ln (1+x)]=0  xln (1+(1/x))−ln (1+x)=0  ln (1+(1/x))^x =ln (1+x)  (1+(1/x))^x =(1+x)  ⇒x=1  f_(max) =f(1)=(ln 2)^2   ⇒f(x)≤(ln 2)^2   for x≠1:  f(x)<(ln 2)^2
$${let}\:{x}=\frac{{a}}{{b}}\neq\mathrm{1} \\ $$$${f}\left({x}\right)=\mathrm{ln}\:\left(\mathrm{1}+{x}\right)×\mathrm{ln}\:\left(\mathrm{1}+\frac{\mathrm{1}}{{x}}\right) \\ $$$${f}'\left({x}\right)=\frac{\mathrm{1}}{\mathrm{1}+{x}}\mathrm{ln}\:\left(\mathrm{1}+\frac{\mathrm{1}}{{x}}\right)−\frac{\mathrm{1}}{{x}^{\mathrm{2}} }×\frac{\mathrm{1}}{\mathrm{1}+\frac{\mathrm{1}}{{x}}}\mathrm{ln}\:\left(\mathrm{1}+{x}\right) \\ $$$${f}'\left({x}\right)=\frac{\mathrm{1}}{{x}\left(\mathrm{1}+{x}\right)}\left[{x}\mathrm{ln}\:\left(\mathrm{1}+\frac{\mathrm{1}}{{x}}\right)−\mathrm{ln}\:\left(\mathrm{1}+{x}\right)\right]=\mathrm{0} \\ $$$${x}\mathrm{ln}\:\left(\mathrm{1}+\frac{\mathrm{1}}{{x}}\right)−\mathrm{ln}\:\left(\mathrm{1}+{x}\right)=\mathrm{0} \\ $$$$\mathrm{ln}\:\left(\mathrm{1}+\frac{\mathrm{1}}{{x}}\right)^{{x}} =\mathrm{ln}\:\left(\mathrm{1}+{x}\right) \\ $$$$\left(\mathrm{1}+\frac{\mathrm{1}}{{x}}\right)^{{x}} =\left(\mathrm{1}+{x}\right) \\ $$$$\Rightarrow{x}=\mathrm{1} \\ $$$${f}_{{max}} ={f}\left(\mathrm{1}\right)=\left(\mathrm{ln}\:\mathrm{2}\right)^{\mathrm{2}} \\ $$$$\Rightarrow{f}\left({x}\right)\leqslant\left(\mathrm{ln}\:\mathrm{2}\right)^{\mathrm{2}} \\ $$$${for}\:{x}\neq\mathrm{1}: \\ $$$${f}\left({x}\right)<\left(\mathrm{ln}\:\mathrm{2}\right)^{\mathrm{2}} \\ $$

Leave a Reply

Your email address will not be published. Required fields are marked *